Difference between revisions of "2017 AMC 10A Problems/Problem 24"

(Solution 1)
(Solution 1.1)
 
(31 intermediate revisions by 8 users not shown)
Line 3: Line 3:
  
 
<math>\textbf{(A)}\ -9009 \qquad\textbf{(B)}\ -8008 \qquad\textbf{(C)}\ -7007 \qquad\textbf{(D)}\ -6006 \qquad\textbf{(E)}\ -5005</math>
 
<math>\textbf{(A)}\ -9009 \qquad\textbf{(B)}\ -8008 \qquad\textbf{(C)}\ -7007 \qquad\textbf{(D)}\ -6006 \qquad\textbf{(E)}\ -5005</math>
 
== Video Solution ==
 
https://youtu.be/3dfbWzOfJAI?t=4412
 
 
~ pi_is_3.14
 
  
 
==Solution 1==
 
==Solution 1==
 
<math>f(x)</math> must have four roots, three of which are roots of <math>g(x)</math>. Using the fact that every polynomial has a unique factorization into its roots, and since the leading coefficient of <math>f(x)</math> and <math>g(x)</math> are the same, we know that
 
<math>f(x)</math> must have four roots, three of which are roots of <math>g(x)</math>. Using the fact that every polynomial has a unique factorization into its roots, and since the leading coefficient of <math>f(x)</math> and <math>g(x)</math> are the same, we know that
  
<cmath>f(x)=g(x)(x-r)</cmath>
+
<cmath>f(x)=g(x)(x+p)</cmath>
  
where <math>r\in\mathbb{C}</math> is the fourth root of <math>f(x)</math>. Substituting <math>g(x)</math> and expanding, we find that
+
where <math>-p\in\mathbb{R}</math> is the fourth root of <math>f(x)</math>.  
 +
(Using <math>(x+p) = (x-r))</math> instead of <math>(x-r)</math> makes the following computations less messy.)
 +
Substituting <math>g(x)</math> and expanding, we find that
  
<cmath>\begin{align*}f(x)&=(x^3+ax^2+x+10)(x-r)\\
+
<cmath>\begin{align*}f(x)&=(x^3+ax^2+x+10)(x+p)\\
&=x^4+(a-r)x^3+(1-ar)x^2+(10-r)x-10r.\end{align*}</cmath>
+
&=x^4+(a+p)x^3+(1+ap)x^2+(10+p)x+10r.\end{align*}</cmath>
  
 
Comparing coefficients with <math>f(x)</math>, we see that
 
Comparing coefficients with <math>f(x)</math>, we see that
  
 
<cmath>\begin{align*}
 
<cmath>\begin{align*}
a-r&=1\\
+
a+p&=1\\
1-ar&=b\\
+
1+ap=b\\
10-r&=100\\
+
10+p&=100\\
-10r&=c.\\
+
10p&=c.\\
 
\end{align*}</cmath>
 
\end{align*}</cmath>
  
(Solution 1.1 picks up here.)
 
  
Let's solve for <math>a,b,c,</math> and <math>r</math>. Since <math>10-r=100</math>, <math>r=-90</math>, so <math>c=(-10)(-90)=900</math>. Since <math>a-r=1</math>, <math>a=-89</math>, and <math>b=1-ar=-8009</math>. Thus, we know that
+
Let's solve for <math>a,b,c,</math> and <math>p</math>. Since <math>10+p=100</math>, <math>p=90</math>.
 +
 
 +
Since <math>a+p=1</math>, <math>a=-89</math>.
 +
 
 +
(Solution 1.1 branches from here and takes a shortcut.)
 +
 
 +
<math>c=(10)(90)=900</math>.
 +
 
 +
Then, since <math>b=1+ap</math>, <math>b=-8009</math>. Thus,
  
 
<cmath>f(x)=x^4+x^3-8009x^2+100x+900.</cmath>
 
<cmath>f(x)=x^4+x^3-8009x^2+100x+900.</cmath>
 +
 +
(Solution 1.2 branches from here and takes another shortcut)
  
 
Taking <math>f(1)</math>, we find that
 
Taking <math>f(1)</math>, we find that
Line 42: Line 48:
 
\end{align*}</cmath>
 
\end{align*}</cmath>
  
==Solution 1.1==
+
===Solution 1.1===
 
A faster ending to Solution 1 is as follows.
 
A faster ending to Solution 1 is as follows.
We shall solve for only <math>a</math> and <math>r</math>. Since <math>10-r=100</math>, <math>r=-90</math>, and since <math>a-r=1</math>, <math>a=-89</math>. Then,
+
 
 
<cmath>\begin{align*}
 
<cmath>\begin{align*}
f(1)&=(1-r)(1^3+a\cdot1^2+1+10)\\
+
f(1)&=(1+p)(1^3+a\cdot1^2+1+10)\\
 
&=(91)(-77)\\
 
&=(91)(-77)\\
 +
&= (7)(13)(11)(-7) = (1001)(-7) \\
 
&=\boxed{\bold{(C)}\, -7007}.\\
 
&=\boxed{\bold{(C)}\, -7007}.\\
 
\end{align*}</cmath>
 
\end{align*}</cmath>
  
==Solution 1.2 (Fast)==
+
===Solution 1.2===
Let the term <math>q(x)</math> be the linear term that we are solving for in the equation <math>f(x)=g(x)\cdot q(x)</math>. Now, we know that <math>q(x)=mx-r</math> must have <math>m=1</math>, because only <math>x \cdot x^3=x^4</math>. In addition, we know that, by distributing, <math>10x-rx=100x</math>. Therefore, <math>r=-90</math>, and all the other variables are quickly solved for.
+
Also a faster ending to Solution 1 is as follows.
 +
 
 +
To find <math>f(1)</math> we just need to find the sum of the coefficients which is <math>1 + 1 - 8009 + 100 + 900= \boxed{\bold{(C)} \ , -7007}.</math>
 +
 
 +
~[[User: Yiyj1|Yiyj1]]
  
 
==Solution 2==
 
==Solution 2==
Line 138: Line 149:
 
Simple polynomial division is a feasible method. Even though we have variables, we can equate terms at the end of the division so that we can cancel terms. Doing the division of <math>\frac{f(x)}{g(x)}</math> eventually brings us the final step <math>(1-a)x^3 + (b-1)x^2 + 90x + c</math> minus <math>(1-a)x^3 - (a-a^2)x^2 + (1-a)x + 10(1-a)</math> after we multiply <math>f(x)</math> by <math>(1-a)</math>. Now we equate coefficients of same-degree <math>x</math> terms. This gives us <math> 10(1-a) = c, b-1 = a - a^2, 1-a = 90 \Rightarrow a = -89, c = 900, b = -8009</math>. We are interested in finding <math>f(1)</math>, which equals <math>1^4 + 1^3 -8009\cdot1^2 + 100\cdot1 + 900 = \boxed{\textbf{(C)}\,-7007}</math>. ~skyscraper
 
Simple polynomial division is a feasible method. Even though we have variables, we can equate terms at the end of the division so that we can cancel terms. Doing the division of <math>\frac{f(x)}{g(x)}</math> eventually brings us the final step <math>(1-a)x^3 + (b-1)x^2 + 90x + c</math> minus <math>(1-a)x^3 - (a-a^2)x^2 + (1-a)x + 10(1-a)</math> after we multiply <math>f(x)</math> by <math>(1-a)</math>. Now we equate coefficients of same-degree <math>x</math> terms. This gives us <math> 10(1-a) = c, b-1 = a - a^2, 1-a = 90 \Rightarrow a = -89, c = 900, b = -8009</math>. We are interested in finding <math>f(1)</math>, which equals <math>1^4 + 1^3 -8009\cdot1^2 + 100\cdot1 + 900 = \boxed{\textbf{(C)}\,-7007}</math>. ~skyscraper
  
==General Note==
+
==Solution 7==
 +
We first note that <math>f(x) = g(x) \cdot q(x) + r(x)</math> where <math>q</math> is the quotient function and <math>r</math> is the remainder function.
  
Note that <math>f(1)</math> for any polynomial is simply the sum of the coefficients of the polynomial.
+
Clearly, <math>r(x) = 0</math> because every single root in <math>g</math> is also in <math>f</math>, thus implying <math>g</math> divides <math>f</math>.
 +
So, we wish to find <math>f(1) = g(1) \cdot q(1)</math>.
 +
 
 +
Such an expression for <math>g(1)</math> is pretty clean here as we can obtain <math>g(1) = a + 12</math>, so we rewrite <math>f(1) = (a + 12) \cdot q(1)</math>.
 +
Well, now we need to know how <math>q</math> is expressed in order to obtain <math>q(1)</math>. This motivates us to long divide to obtain the quotient function. After simple long division <math>q(x) = x + (1 - a)</math>. In addition, what is left over, namely <math>r(x)</math>, has a constant piece of <math>a + 89</math> (you'll see in a few sentences why we only care about particularly the constant piece).
 +
 
 +
Now we can write: <math>f(1) = (a + 12) \cdot (2 - a)</math>.
 +
 
 +
Now, as we have already established <math>r(x) = 0</math> for ALL <math>x</math> that means <math>r(0)</math> or the constant piece is <math>0</math>, so <math>89 + a = 0</math>, in which we obtain <math>a = -89</math>. We now plug this back into our equation for <math>f(1)</math> to get <math>(-89 + 12)(2 - (89)) = -77 \cdot 91 = \boxed{\textbf{(C)}\,-7007}</math>. ~triggod
 +
 
 +
==General Notes==
 +
<math>f(1)</math> for any polynomial is simply the sum of the coefficients of the polynomial.
 +
 
 +
<math>f(x)/g(x) = x+r</math> must have real <math>r</math>. Both <math>f</math> and <math>g</math> have all real coefficients, and so odd-degree <math>g</math> must have an odd number of real roots, and even-degree <math>f</math> must have an even number of real roots, so <math>f</math>'s single additional root must be real.
 +
 
 +
 
 +
<math>77 * 91 = 7 * 11 * 7 * 13</math>, and <math>7*11*13=1001</math> is a good number sense fact to know. It's interesting because <math>1001= 10^3+1</math> and the 3 nearest primes to <math> 10 </math> are <math>7,11,13</math>.
  
 
==Video Solution==
 
==Video Solution==
Line 146: Line 174:
  
 
https://www.youtube.com/watch?v=MBIiz0mroqk  (By Richard Rusczyk)
 
https://www.youtube.com/watch?v=MBIiz0mroqk  (By Richard Rusczyk)
 +
 +
https://youtu.be/3dfbWzOfJAI?t=4412
 +
 +
~ pi_is_3.14
  
 
==See Also==
 
==See Also==

Latest revision as of 23:16, 7 October 2023

Problem

For certain real numbers $a$, $b$, and $c$, the polynomial \[g(x) = x^3 + ax^2 + x + 10\]has three distinct roots, and each root of $g(x)$ is also a root of the polynomial \[f(x) = x^4 + x^3 + bx^2 + 100x + c.\]What is $f(1)$?

$\textbf{(A)}\ -9009 \qquad\textbf{(B)}\ -8008 \qquad\textbf{(C)}\ -7007 \qquad\textbf{(D)}\ -6006 \qquad\textbf{(E)}\ -5005$

Solution 1

$f(x)$ must have four roots, three of which are roots of $g(x)$. Using the fact that every polynomial has a unique factorization into its roots, and since the leading coefficient of $f(x)$ and $g(x)$ are the same, we know that

\[f(x)=g(x)(x+p)\]

where $-p\in\mathbb{R}$ is the fourth root of $f(x)$. (Using $(x+p) = (x-r))$ instead of $(x-r)$ makes the following computations less messy.) Substituting $g(x)$ and expanding, we find that

\begin{align*}f(x)&=(x^3+ax^2+x+10)(x+p)\\ &=x^4+(a+p)x^3+(1+ap)x^2+(10+p)x+10r.\end{align*}

Comparing coefficients with $f(x)$, we see that

\begin{align*} a+p&=1\\ 1+ap=b\\ 10+p&=100\\ 10p&=c.\\ \end{align*}


Let's solve for $a,b,c,$ and $p$. Since $10+p=100$, $p=90$.

Since $a+p=1$, $a=-89$.

(Solution 1.1 branches from here and takes a shortcut.)

$c=(10)(90)=900$.

Then, since $b=1+ap$, $b=-8009$. Thus,

\[f(x)=x^4+x^3-8009x^2+100x+900.\]

(Solution 1.2 branches from here and takes another shortcut)

Taking $f(1)$, we find that

\begin{align*} f(1)&=1^4+1^3-8009(1)^2+100(1)+900\\ &=1+1-8009+100+900\\ &=\boxed{\bold{(C)}\, -7007}.\\ \end{align*}

Solution 1.1

A faster ending to Solution 1 is as follows.

\begin{align*} f(1)&=(1+p)(1^3+a\cdot1^2+1+10)\\ &=(91)(-77)\\ &= (7)(13)(11)(-7) = (1001)(-7) \\ &=\boxed{\bold{(C)}\, -7007}.\\ \end{align*}

Solution 1.2

Also a faster ending to Solution 1 is as follows.

To find $f(1)$ we just need to find the sum of the coefficients which is $1 + 1 - 8009 + 100 + 900= \boxed{\bold{(C)} \ , -7007}.$

~Yiyj1

Solution 2

We notice that the constant term of $f(x)=c$ and the constant term in $g(x)=10$. Because $f(x)$ can be factored as $g(x) \cdot (x- r)$ (where $r$ is the unshared root of $f(x)$, we see that using the constant term, $-10 \cdot r = c$ and therefore $r = -\frac{c}{10}$. Now we once again write $f(x)$ out in factored form:

\[f(x) = g(x)\cdot (x-r) = (x^3+ax^2+x+10)(x+\frac{c}{10})\].

We can expand the expression on the right-hand side to get:

\[f(x) = x^4+(a+\frac{c}{10})x^3+(1+\frac{ac}{10})x^2+(10+\frac{c}{10})x+c\]

Now we have $f(x) = x^4+(a+\frac{c}{10})x^3+(1+\frac{ac}{10})x^2+(10+\frac{c}{10})x+c=x^4+x^3+bx^2+100x+c$.

Simply looking at the coefficients for each corresponding term (knowing that they must be equal), we have the equations: \[10+\frac{c}{10}=100 \Rightarrow c=900\] \[a+\frac{c}{10} = 1, c=900 \Rightarrow a + 90 =1 \Rightarrow a= -89\]

and finally,

\[1+\frac{ac}{10} = b = 1+\frac{-89 \cdot 900}{10} = b = -8009\].

We know that $f(1)$ is the sum of its coefficients, hence $1+1+b+100+c$. We substitute the values we obtained for $b$ and $c$ into this expression to get $f(1) = 1 + 1 + (-8009) + 100 + 900 = \boxed{\textbf{(C)}\,-7007}$.

Solution 3

Let $r_1,r_2,$ and $r_3$ be the roots of $g(x)$. Let $r_4$ be the additional root of $f(x)$. Then from Vieta's formulas on the quadratic term of $g(x)$ and the cubic term of $f(x)$, we obtain the following:

\begin{align*} r_1+r_2+r_3&=-a \\  r_1+r_2+r_3+r_4&=-1 \end{align*}

Thus $r_4=a-1$.

Now applying Vieta's formulas on the constant term of $g(x)$, the linear term of $g(x)$, and the linear term of $f(x)$, we obtain:

\begin{align*} r_1r_2r_3  & = -10\\ r_1r_2+r_2r_3+r_3r_1 &= 1\\  r_1r_2r_3+r_2r_3r_4+r_3r_4r_1+r_4r_1r_2  & = -100\\ \end{align*}

Substituting for $r_1r_2r_3$ in the bottom equation and factoring the remainder of the expression, we obtain:

\[-10+(r_1r_2+r_2r_3+r_3r_1)r_4=-10+r_4=-100\]

It follows that $r_4=-90$. But $r_4=a-1$ so $a=-89$

Now we can factor $f(x)$ in terms of $g(x)$ as

\[f(x)=(x-r_4)g(x)=(x+90)g(x)\]

Then $f(1)=91g(1)$ and

\[g(1)=1^3-89\cdot 1^2+1+10=-77\]

Hence $f(1)=91\cdot(-77)=\boxed{\textbf{(C)}\,-7007}$.

Solution 4 (Risky)

Let the roots of $g(x)$ be $r_1$, $r_2$, and $r_3$. Let the roots of $f(x)$ be $r_1$, $r_2$, $r_3$, and $r_4$. From Vieta's, we have: \begin{align*} r_1+r_2+r_3=-a \\ r_1+r_2+r_3+r_4=-1 \\ r_4=a-1 \end{align*} The fourth root is $a-1$. Since $r_1$, $r_2$, and $r_3$ are common roots, we have: \begin{align*} f(x)=g(x)(x-(a-1)) \\ f(1)=g(1)(1-(a-1)) \\ f(1)=(a+12)(2-a) \\ f(1)=-(a+12)(a-2) \\ \end{align*} Let $a-2=k$: \begin{align*} f(1)=-k(k+14) \end{align*} Note that $-7007=-1001\cdot(7)=-(7\cdot(11)\cdot(13))\cdot(7)=-91\cdot(77)$ This gives us a pretty good guess of $\boxed{\textbf{(C)}\, -7007}$.

Solution 5

First off, let's get rid of the $x^4$ term by finding $h(x)=f(x)-xg(x)$. This polynomial consists of the difference of two polynomials with $3$ common factors, so it must also have these factors. The polynomial is $h(x)=(1-a)x^3 + (b-1)x^2 + 90x + c$, and must be equal to $(1-a)g(x)$. Equating the coefficients, we get $3$ equations. We will tackle the situation one equation at a time, starting the $x$ terms. Looking at the coefficients, we get $\dfrac{90}{1-a} = 1$. \[\therefore 90=1-a.\] The solution to the previous is obviously $a=-89$. We can now find $b$ and $c$. $\dfrac{b-1}{1-a} = a$, \[\therefore b-1=a(1-a)=-89*90=-8010\] and $b=-8009$. Finally $\dfrac{c}{1-a} = 10$, \[\therefore c=10(1-a)=10*90=900\] Solving the original problem, $f(1)=1 + 1 + b + 100 + c = 102+b+c=102+900-8009=\boxed{\textbf{(C)}\, -7007}$.

Solution 6

Simple polynomial division is a feasible method. Even though we have variables, we can equate terms at the end of the division so that we can cancel terms. Doing the division of $\frac{f(x)}{g(x)}$ eventually brings us the final step $(1-a)x^3 + (b-1)x^2 + 90x + c$ minus $(1-a)x^3 - (a-a^2)x^2 + (1-a)x + 10(1-a)$ after we multiply $f(x)$ by $(1-a)$. Now we equate coefficients of same-degree $x$ terms. This gives us $10(1-a) = c, b-1 = a - a^2, 1-a = 90 \Rightarrow a = -89, c = 900, b = -8009$. We are interested in finding $f(1)$, which equals $1^4 + 1^3 -8009\cdot1^2 + 100\cdot1 + 900 = \boxed{\textbf{(C)}\,-7007}$. ~skyscraper

Solution 7

We first note that $f(x) = g(x) \cdot q(x) + r(x)$ where $q$ is the quotient function and $r$ is the remainder function.

Clearly, $r(x) = 0$ because every single root in $g$ is also in $f$, thus implying $g$ divides $f$. So, we wish to find $f(1) = g(1) \cdot q(1)$.

Such an expression for $g(1)$ is pretty clean here as we can obtain $g(1) = a + 12$, so we rewrite $f(1) = (a + 12) \cdot q(1)$. Well, now we need to know how $q$ is expressed in order to obtain $q(1)$. This motivates us to long divide to obtain the quotient function. After simple long division $q(x) = x + (1 - a)$. In addition, what is left over, namely $r(x)$, has a constant piece of $a + 89$ (you'll see in a few sentences why we only care about particularly the constant piece).

Now we can write: $f(1) = (a + 12) \cdot (2 - a)$.

Now, as we have already established $r(x) = 0$ for ALL $x$ that means $r(0)$ or the constant piece is $0$, so $89 + a = 0$, in which we obtain $a = -89$. We now plug this back into our equation for $f(1)$ to get $(-89 + 12)(2 - (89)) = -77 \cdot 91 = \boxed{\textbf{(C)}\,-7007}$. ~triggod

General Notes

$f(1)$ for any polynomial is simply the sum of the coefficients of the polynomial.

$f(x)/g(x) = x+r$ must have real $r$. Both $f$ and $g$ have all real coefficients, and so odd-degree $g$ must have an odd number of real roots, and even-degree $f$ must have an even number of real roots, so $f$'s single additional root must be real.


$77 * 91 = 7 * 11 * 7 * 13$, and $7*11*13=1001$ is a good number sense fact to know. It's interesting because $1001= 10^3+1$ and the 3 nearest primes to $10$ are $7,11,13$.

Video Solution

https://youtu.be/wXvRNC-48Lk

https://www.youtube.com/watch?v=MBIiz0mroqk (By Richard Rusczyk)

https://youtu.be/3dfbWzOfJAI?t=4412

~ pi_is_3.14

See Also

2017 AMC 10A (ProblemsAnswer KeyResources)
Preceded by
Problem 23
Followed by
Problem 25
1 2 3 4 5 6 7 8 9 10 11 12 13 14 15 16 17 18 19 20 21 22 23 24 25
All AMC 10 Problems and Solutions
2017 AMC 12A (ProblemsAnswer KeyResources)
Preceded by
Problem 22
Followed by
Problem 24
1 2 3 4 5 6 7 8 9 10 11 12 13 14 15 16 17 18 19 20 21 22 23 24 25
All AMC 12 Problems and Solutions

The problems on this page are copyrighted by the Mathematical Association of America's American Mathematics Competitions. AMC logo.png